Solutions to Exercises in Real
Analysis
Li Gongbao
School of Mathematics and Statistics in CCNU
PREFACE
The Preface can be written here
CHAPTER
ONE
1. Does there exist an infinite σ-algebra which has only countably many
members?
Solution: No. There exists no such an infinite σ−algebra M which has only
countably members.
T Aj = ∅(i 6= j), then
Proof 1: Claim 1: If X = Sn
i=1 Ai, Ai 6= ∅, Ai ∈ M, Ai
there exists a A ∈ M, A 6= ∅, s.t. AT Aj 6= ∅ and AT Ac
j 6= ∅ for some j ∈
{1, 2,··· , n}.Hence, Aj = (AT Aj)S(AT Ac
j ∈ M,
then X can be written by Sn+1
i=1 Bj,∅ 6= Bj ∈ M,and Bj are pairwise disjoint.
Proof of Claim 1: Put F{A1, A2,··· , An} be the σ-algebra spaned by
A1, A2,··· , An.Since Ai are pairwise disjoint,new numbers different from Ai
can be only producted by union of some Ai’s.However,{S
j∈{1,2,··· ,n} Aj} has at
most 2n numbers.So F{A1, A2,··· , An} is finite.
AT Aj 6= ∅ and AT Ac
implies that AcT Aj = ∅,thus A ⊆ Aj,which contains the relation:
By the definition of M,there exists A ∈ M s.t.A 6∈ F.We claim that
j 6= ∅ for some j ∈ {1, 2,··· , n}.If not,then AT Aj 6= ∅
j) and AT Aj ∈ M, AT Ac
Aj ⊆ A.
[
{j;AT Aj6=∅}
In fact,the above inclusion relation is mutual,for if x ∈ A ⊆ X,then x ∈ Aj
for some j,and AT Aj
6= ∅,so x ∈ S{j;AT Aj6=∅} Aj,which proves that A ⊆
S{j;AT Aj6=∅} Aj.Hence A = S{j;AT Aj6=∅} Aj ∈ F.A contradiction.This com-
def= A1S Ac1.By Claim 1,there exists a C1 ∈ M s.t.C1T Ai 6=
X,then X = A1S A2
6= ∅ for some i,say,i = 1.So A1 is the union of C1S A1 and
∅ and C c1
Now we prove exercise 1.Choose A1 be a nonempty proper subset of
pletes the proof.
T Ai
4
C c1
S A1,both of which are nonempty measurable subsets. Thus
by rearranging the subscripts we can write X = A1S A2S A3,where Ai(i =
Ai)[(C c
A1)[
X = (C1
\
\
ABSTRACT INTEGRATION
1
A2,
1, 2, 3)are nonempty measurable proper subsets of X which are mutually
pairwise disjoint.Continuing in this way, we get a countably infinite disjoint
collection {Ai}∞
T Aj = ∅ for i 6= j.
Put M1 = {Ai}∞
i=1,then M1 ⊆ M.Since M is a σ−algebra and M1 is in-
finitely countable,2M1 is a subcollection of M.Hence the cardinality inequality
holds:
i=1 with Ai
2M1 ≤ 2M,
which is a contradiction since the former cardinality is equal to [0, 1] ,greater
than Nwhich is the latter one’s cardinality.(See P25, exercise 3 in the book
of Jiang zejian). We have completed the proof 1.
Proof 2: We give another way to find a countable subset of M by the im-
plication of proof 1, the main tool is Zorn’s lemma(See the remark). Since
M is countable we denote M by {Ai}∞
i=1.Then M is partially ordered by the
set inclusion relation ⊆.
If all totally ordered subsets of M are finite,then X,the whole space,is ob-
viously their common upper bound.Put B0 = X and M1 = M−{B0},apparently
that all totally ordered subsets of M1 is finite,hence their exists a maximal
element B1 ∈ M1 by Zorn’s lemma.
Put
M2 = {Ai|Ai ∈ M, Ai B1} ⊆ M1
T Bc1 6=
We claim that M2 is infinite.If not,then put C = {Ai|Ai ∈ M1, Ai
∅ and Ai
6= Bc1},is infinite,which will lead to a result that the infinite set
{Bi − Ai|Ai ∈ C} ⊆ M2,a contradiction to our assumption.To see this, pick
S B1 ∈ M1
an arbitrary Ai ∈ C,then B1 ⊆ Ai
S B1 since B1 is the maximal element in M1,which implies that
and B1 = Ai
S B1 = X,by this,∅ 6=
Ai ⊆ B1that contradicts to the definition of C.So Ai
B1 − Ai B1.For if B1 − Ai = ∅,then B1 Ai which implies that Ai =
S B1 = X,contradiction to Ai 6= X since Ai ∈ M1.IfB1 − Ai = B1,then
Ai
S B1 6= X,then Ai
S B1.If Ai
REAL AND COMPLEX ANALYSIS
A1 = ∅ or Ai Bc1.Therefore the set
{Bi − Ai|Ai ∈ C} ⊆ M2
5
We have proved that M2 is infinite in the above.Applying Zorn’s lemma
to M2 ,there exists a maximal element B2 ∈ M2 with B2 B1.Put
M3 = {Ai|Ai ∈ M, Ai B2} ⊆ M2.
Then M3 is infinite if we replace X by B1 and do the same as above.Hence
there is a maximal element B3 ∈ M3 with B3 B2.Continuing in this way
and we get a countably infinite collection {Bi}∞
i=1 with the property
B0 B1 B2 B3 ···
Contradiction to our assumption of M that M has no infinite totally ordered
subset.
Hence M is bounded to have a totally ordered subset,which we denote
(1)If ˜M1 has no upper bound,then there exists a subsequence {Cni} ⊆
by ˜M1 = {Ci}∞
i=1 ⊆ M.
{Ci} s.t.
∅ = Ci0
Ci1
Ci3
··· Cik
Ci2
k=1 ⊆ M is pairwise disjoint hence is just what we
Cik+1
···
Then the set {Cik+1 − Cik}∞
want to find.
k=1 has the property :
··· Dik
Dik+1
···
(2)If ˜M1 has a upper bound D1,then put ˜M2 = ˜M1 − {D1}.Continuing
in this way,if for some k, ˜Mk = ˜Mk−1 − {Dk−1} has no upper bound,we have
completed the proof; or else,then {Dk}∞
∅ = Di0
Di3
Di2
k=1 ⊆ M is just what we want,and we have completed
Di1
and the set {Dk+1−Dk}∞
the proof 2.
Remark:1. We give the proof that 2N = [0, 1] = C here. For any giv-
en A ⊂ N, set ϕA(n) = { 1,
Let g : A → (0, 1) is given by
0,
g(A) = 0. ϕA(1)ϕA(2) · ··, it is obvious that g is one-to-one. So, 2N ≤ [0, 1].
if n ∈ A;
if n 6∈ A.
6
ABSTRACT INTEGRATION
On the other hand, for any x ∈ (0, 1), set Ax = {r; r ≤ x, r ∈ R}, where
R is the set of all the rational in (0, 1). If x, y ∈ (0, 1) and x 6= y, by Berstein’s
theorem, the density of rational in R, Ax 6= Ay. So, (0, 1) ≤ R = 2N.
So, 2N = [0, 1] = C.
2.Zorn’s lemma:Suppose (X,≺) is a partially ordered set.If every totally or-
dered subsets of X has one upper bound,then there exists a maximal element
a ∈ X s.t.for anyx ∈ X,if x ≺ a,then x = a.
2
2. Prove an analogue of Theorem 1.8 for n functions.
Solution: We have the following result.
Theorem 1.80: Let u1, u2, ..., un be real measurable functions on a measurable
space X, let Φ be a continuous mapping of Rn onto a topological space Y ,
and define
h(x) = Φ(u1(x), ..., un(x))
for x ∈ X. Then h : X → Y is measurable.
Proof: Since h = Φ ◦ f, where f(x) = (u1(x), ..., un(x)) and f maps X into
Rn. Theorem 1.7 shows that it is enough to prove the measurability of f.
If R is any open rectangle in Rn with sides parallel to the coordinate
super plane, then R = I1×I2×···×In, where Ii = (ai, bi) ⊂ R1 (i = 1, 2, ..., n)
are open segments and
f−1(R) = u−1
1 (I1)\
2 (I2)\· · ·\
u−1
u−1
n (In).
In fact, for any given x ∈ f−1(R), we have f(x) = (u1(x), ..., un(x)) ∈
i (Ii) for i = 1, 2, ..., n,
i (Ii), we have
R = I1 × I2 × · · · × In and ui(x) ∈ Ii, then x ∈ u−1
i (Ii). On the other hand, for any x ∈ Tn
that is x ∈ Tn
xi ∈ u−1
So f−1(R) is measurable for each open rectangle interval R ⊂ Rn as ui are
measurable on X.
i (Ii), ui(x) ∈ Ii and f(x) = (u1(x), ..., un(x)) ∈ I1 × I2 × · · · × In.
i=1 u−1
i=1 u−1
By Lindel˝of’s Theorem, every open set V is a countable union of open
interval with sides parallel to the coordinate super-plane, V = S Ri, So
f−1(V ) = Sn
2
3. Prove that if f is a real function on a measurable space X such that
{x; f(x) ≥ r} is measurable for every rational r, then f if measurable.
i=1 f−1(Ri) is measurable as well.
In fact, for any x ∈ {x; f(x) > α}, there is a n0 such that α < rn0 ≤ f(x),
therefore x ∈ {x; f(x) > α}. On the other hand, rn > α implies that
+∞[
n=1
{x; f(x) ≥ rn} ⊂ {x; f(x) > α}.
REAL AND COMPLEX ANALYSIS
7
Proof: For any α ∈ R1, there is a sequence of rational {rn; n = 1, 2, ...} such
that r1 > r2 > r3 > ... > rn > ... > α with lim
{x; f(x) > α} =
n→∞ rn = α. So
{x; f(x) ≥ rn}.
+∞[
n=1
By the condition, for any given n, {x; f(x) ≥ rn} is measurable, we see
that {x; f(x) > α} is measurable for any α ∈ R1. By Theorem 1.12, f is
measurable.
2
4. Let {an} and {bn} be sequence in [−∞, +∞], prove the following asser-
tions:
n→∞ (−an) = − lim inf
(a) lim sup
n→∞ an;
n→∞ (an + bn) ≤ lim sup
(b) lim sup
n→∞ an + lim sup
the form ∞ − ∞;
(c) If an ≤ bn for all n, then
n→∞ bn provided none of the sums is of
n→∞ an ≤ lim inf
lim inf
n→∞ bn
show by an example that strict inequality can hold in (b).
Proof:
(a) lim sup
n→∞ (−an) = inf
k≥1
sup{−ak,−ak+1, ...} = inf
k≥1
inf{ak, ak+1, ...} = − lim inf
{− inf{ak, ak+1, ...}}
n→∞ an.
= − sup
k≥1
(b) lim sup
[sup{ak + bk, ak+1 + bk+1, ...}]
n→∞ (an + bn) = inf
k≥1
k→∞[sup{ak + bk, ak+1 + bk+1, ...}]
= lim
{an} + sup
≤ lim
{bn}]
k→∞[sup
n≥k
n≥k
n→∞ [sup{bn}]
n→∞[sup{an}] + lim inf
= lim
n→∞ an + lim sup
= lim sup
n→∞ bn.
{ak} = lim inf
k→∞ inf
n→∞ an.
n≥k
{an} ≤ inf
(c) lim inf
n→∞ an = lim
In (b), the strict inequality can hold. For example, let an = (−1)n, bn =
k→∞ inf
n≥k
n→∞ bn.
n→∞ an + lim sup
n→∞ an = 1, lim sup
n→∞ bn = 1, but lim sup
8
ABSTRACT INTEGRATION
−(−1)n, then an + bn = 0, lim sup
n→∞ (an +
bn) = 0 < 2 = 1 + 1 = lim sup
2
5. (a) Suppose f : X → [−∞,∞] and g : X → [−∞,∞] are measurable.
Prove that the sets {x; f(x) < g(x)}, {x; f(x) = g(x)} are measurable.
(b) Prove that the set of points at which a sequence of measurable real-valued
functions converges(to a finite limit) is measurable.
Proof: (a1) Suppose f, g are measurable on X, then {x; f(x) < g(x)} is
measurable. Suppose {rn} is the set of rational in R1, we claim that
[{x; f(x) < rn}\{x; rn < g(x)}]
{x; f(x) < g(x)} =
∞[
n=1
In fact,
x0 ∈ {x; f(x) < g(x)} ⇐⇒ f(x0) < g(x0)
⇐⇒ ∃rn0, such that f(x0) < rn0 < g(x0)
⇐⇒ x0 ∈ S∞
n=1[{x; f(x) < rn}T{x; rn < g(x)}].
For any r ∈ R, {x; f(x) < r} and {x; r < g(x)} are measurable s-
ince f, g are measurable on X. So, by the claim we have proved above,
{x; f(x) < g(x)} is measurable.
(a2) By (a1), {x; f(x) = g(x)} = X − [{x; f(x) < g(x)}S{x; f(x) >
(b) Suppose {fn} is a sequence of real-valued measurable functions on
g(x)}] is measurable.
(X, M)(a measurable space), then by Theorem 1.9(c), for any n, m ∈ N, |fn(x)−
fm(x)| is a measurable function as | · | is a continuous function on R1 × R1.
So, {x; |fn(x) − fm(x)| < a} is a measurable set for any a ∈ R1.
Clearly, the set where {fn(x)} has finite limit is given by
}
{x; |fn(x) − fm(x)| <
+∞\
+∞[
A =
\
k=1
N=1
n,m≥N
1
k
by Cauchy’s criterion for convergence.
A is a measurable set because for any n, m ∈ N and k ∈ N, {x; |fn(x)−
k} ∈ M and M is a σ−algebra.
fm(x)| < 1
2
6. Let X be an uncountable set, let M be the collection of all sets E ⊂ X
such that either E or Ec is at most countable, and define µ(E) = 0 in the
first case, µ(E) = 1 in the second. Prove that M is a σ−algebra in X and